Spanning Set of R^3: (1,0,0)^T,(0,1,1)^T,(1,0,1)^T,(1,2,3)^T

  • Thread starter Dustinsfl
  • Start date
  • Tags
    Set
In summary: For example, if you get a row of zeroes (except for the last element) that says v1 = -c2v2 - c3v3 - c4v4, which means you don't need v1 in your basis. You can take any combination of the others and get v1. So you can eliminate v1 from your original list.In this case, when I did the row-reduction, I got the matrix {{1,0,0,0}, {0,1,0,-1}, {0,0,1,1}, {0,0,0,0}} which means that v1 is a linear combination of the other three vectors. So your spanning set is v1, v
  • #1
Dustinsfl
2,281
5
Determine the spanning set of {(1,0,0)^T, (0,1,1)^T, (1,0,1)^T, (1,2,3)^T}

v=a*(1,0,0)^T+b*(0,1,1)^T+c*(1,0,1)^T+d*(1,2,3)^T

Augmented Matrix(3, 5, {(1, 1) = 1, (1, 2) = 0, (1, 3) = 1, (1, 4) = 1, (1, 5) = v1, (2, 1) = 0, (2, 2) = 1, (2, 3) = 0, (2, 4) = 2, (2, 5) = v2, (3, 1) = 0, (3, 2) = 1, (3, 3) = 1, (3, 4) = 3, (3, 5) = v3})

In reduced row echelon, Matrix(3, 5, {(1, 1) = 1, (1, 2) = 0, (1, 3) = 0, (1, 4) = 0, (1, 5) = v1-v3+v2, (2, 1) = 0, (2, 2) = 1, (2, 3) = 0, (2, 4) = 2, (2, 5) = v2, (3, 1) = 0, (3, 2) = 0, (3, 3) = 1, (3, 4) = 1, (3, 5) = v3-v2})

I am not sure how to finish this off since there is an extra column.
 
Physics news on Phys.org
  • #2
Dustinsfl said:
Determine the spanning set of {(1,0,0)^T, (0,1,1)^T, (1,0,1)^T, (1,2,3)^T}

v=a*(1,0,0)^T+b*(0,1,1)^T+c*(1,0,1)^T+d*(1,2,3)^T

Augmented Matrix(3, 5, {(1, 1) = 1, (1, 2) = 0, (1, 3) = 1, (1, 4) = 1, (1, 5) = v1, (2, 1) = 0, (2, 2) = 1, (2, 3) = 0, (2, 4) = 2, (2, 5) = v2, (3, 1) = 0, (3, 2) = 1, (3, 3) = 1, (3, 4) = 3, (3, 5) = v3})

In reduced row echelon, Matrix(3, 5, {(1, 1) = 1, (1, 2) = 0, (1, 3) = 0, (1, 4) = 0, (1, 5) = v1-v3+v2, (2, 1) = 0, (2, 2) = 1, (2, 3) = 0, (2, 4) = 2, (2, 5) = v2, (3, 1) = 0, (3, 2) = 0, (3, 3) = 1, (3, 4) = 1, (3, 5) = v3-v2})

I am not sure how to finish this off since there is an extra column.

Most of the stuff from "Augmented Matrix ..." on down is gobbledygook that I will ignore.

You have a set of vectors in R3 that includes four vectors, so obviously there are too many for a basis. What you want to do is to eliminate any of the four vectors that is a linear combination of the others. To save myself some typing I'm going to call your four vectors v1, v2, v3, and v4. If you put your four vectors as columns in a 4 x 3 (not augmented) matrix and row-reduce it, you are finding the solutions to the equation c1v1 + c2v2 + c3v3 + c4v4 = 0.

From that you should be able to see which vector is a linear combination of the others.
 

1. What is a spanning set?

A spanning set is a set of vectors that can be used to generate all other vectors within a given vector space through linear combinations.

2. How many vectors are in this spanning set?

There are four vectors in this spanning set: (1,0,0)^T, (0,1,1)^T, (1,0,1)^T, and (1,2,3)^T.

3. How do I know if a set of vectors spans a vector space?

A set of vectors spans a vector space if every vector in the space can be written as a linear combination of the vectors in the set.

4. Can this spanning set be used to generate all vectors in R^3?

Yes, this spanning set can be used to generate all vectors in R^3 through linear combinations.

5. How do I find the coefficients for a specific vector in this spanning set?

To find the coefficients for a specific vector, you can set up a system of equations and use Gaussian elimination to solve for the coefficients. Alternatively, you can use the inverse matrix method or the Cramer's rule to solve for the coefficients.

Similar threads

  • Calculus and Beyond Homework Help
Replies
3
Views
330
  • Calculus and Beyond Homework Help
Replies
10
Views
2K
  • Calculus and Beyond Homework Help
Replies
8
Views
2K
  • Calculus and Beyond Homework Help
Replies
8
Views
621
  • Calculus and Beyond Homework Help
Replies
3
Views
570
  • Calculus and Beyond Homework Help
Replies
1
Views
1K
  • Calculus and Beyond Homework Help
Replies
1
Views
1K
  • Calculus and Beyond Homework Help
Replies
2
Views
985
  • Calculus and Beyond Homework Help
Replies
7
Views
1K
  • Calculus and Beyond Homework Help
Replies
4
Views
3K
Back
Top